1
$\begingroup$

I am currently working through the first chapter of Molecular Electronic-Structure Theory (Helgaker et al.). They are demonstrating that, in a complete basis set, commutation relations that hold for operators in their first quantization representation also hold for those operators in their second quantization representation. Using their notation, in first quantization: $$[r_{\alpha}^c, p_{\beta}^c] = i\delta_{\alpha\beta}^c$$ where \begin{align} r_{\alpha}^c&=\sum_{j=1}^N r_{\alpha}^c(j)\\ p_{\alpha}^c&=\sum_{j=1}^N p_{\alpha}^c(j)\\ \delta_{\alpha\beta}^c&=\sum_{j=1}^N \delta_{\alpha\beta}(j) \end{align} and the Greek letters index cartesian directions. The corresponding second quantization representation of $r_{\alpha}^c$, for instance, would be $$\hat{r}_{\alpha}=\sum_{PQ}[r_{\alpha}^c]_{PQ}\,a_{P}^{\dagger}a_{Q}$$ where $a_{P}^{\dagger}$, $a_{Q}$ are the creation/annihilation operators, respectively, and $$[r_{\alpha}^c]_{PQ}=\int\phi_{P}^*(x)\,r_{\alpha}^c\,\phi_{Q}(x)dx$$ for spin orbitals $\phi_P(x)$, $\phi_Q(x)$. My issue is near the last line of their derivation (1.5.31). In a complete basis set, they state that: \begin{align} [\hat{r}_{\alpha}, \hat{p}_{\beta}] &= \sum_{P,Q=1}^{\infty} [[r_{\alpha}^c, p_{\beta}^c]]_{PQ}\,a_{P}^{\dagger}a_{Q}\\ &= \sum_{P,Q=1}^{\infty} [i\delta_{\alpha\beta}^c]_{PQ}\,a_{P}^{\dagger}a_{Q}\\ &= i\delta_{\alpha\beta}\hat{N} \end{align} where $\hat{N} = \sum_Pa_{P}^{\dagger}a_{P}$ is the number operator. These last two lines are giving me trouble. I understand that a $\delta_{PQ}$ comes out of $[\delta_{\alpha\beta}^c]_{PQ}$ due to the orthonormality of the spin orbitals, which collapses the P,Q sum into $\hat{N}$. However, I do not understand how $\delta_{\alpha\beta}^c$ becomes $\delta_{\alpha\beta}$, as $\delta_{\alpha\beta}^c$ is not at all affected by the integral and should just be able to be pulled out entirely (this is what happens to get the $\delta_{PQ}$ in the first place!). I feel like I am missing something very basic.

Thanks!

$\endgroup$

1 Answer 1

1
$\begingroup$

It need not be affected by the integral. If the cartesian directions do not match, the commutator inside the sum vanishes, and so does the entire r.h.s. So the $\delta_{\alpha\beta}$ is justified, since the l.h.s. is zero unless the Cartesian subscripts match.

$\endgroup$
8
  • 1
    $\begingroup$ @frobenius Hello, mate. I'm sorry about my answer not being complete. Let's see if this helps. The second-quantized operators you are considering here are symmetric one particle operators, since they are a sum of operators each acting on the single-particle Hilbert of a given particle, with the same phase. The matrix elements appearing in the second-quantized expansion of the operator are taken in the single-particle space, and hence the commutator whose matrix element you have inside the sum is referred to a single-particle. No sum over particles. You may check how these are built in Landau's $\endgroup$ Commented Dec 30, 2024 at 19:27
  • 1
    $\begingroup$ In Landau's book on Non-relativistic Quantum Mechanics, I mean. This is the same for two-particle operators. For bosons, the matrix element multiplying the creation/annihilation operators in the expansion would be taken in the two-particle Hilbert factor of the Fock space, for example. In your example, It's a matrix element on single-particle orbitals. Here, the problem is the notation is a little bit confusing, since you use the same symbol for the single-particle position operator and for the symmetric sum of position operators of the $N$ particles. $\endgroup$ Commented Dec 30, 2024 at 19:29
  • 1
    $\begingroup$ Just for completion allow me to give you a glimpse of where the matrix elements come from. In a theory built on vectors of identical particles living in Fock's space, one may take a symmetric sum of the single-particle operators as you did. One may transform the basis of orbitals chosen and its creation/annihilation operators to the basis of orbitals that diagonalize the single-particle operator in the one-particle space. This way one realizes that the matrix elements of the symmetric sum of single-particle operators is just the sum over "eigenorbitals" of the corresponding eigenvalues times $\endgroup$ Commented Dec 30, 2024 at 19:39
  • 1
    $\begingroup$ the occupation number. Going back to the original basis of orbitals reveals the second-quantized expression you brought, and the matrix element over single-particle orbitals emerges naturally. This is the origin of it. In the process one changes the sum over particles to a sum over orbitals. $\endgroup$ Commented Dec 30, 2024 at 19:40
  • 1
    $\begingroup$ @frobenius I agree. It is strange. Maybe it is a typo. $\endgroup$ Commented Dec 30, 2024 at 20:46

Your Answer

By clicking “Post Your Answer”, you agree to our terms of service and acknowledge you have read our privacy policy.

Start asking to get answers

Find the answer to your question by asking.

Ask question

Explore related questions

See similar questions with these tags.